PT1.S3.Q10 - Even in a democracy, it is necessary

nc______nc______ Alum Member
edited March 2018 in Logical Reasoning 42 karma

I understand that (c) is a better answer choice than (b), but I couldn't rule (b) out either. Isn't the direction of public policy out of scope with regards to the stimulus? Is there something in the passage that I am missing here? Many thanks in advance!

Admin note: edited title

Comments

  • FixedDiceFixedDice Member
    edited February 2018 1804 karma

    The problem with (B) is that the argument simply doesn't make the said drastic assumption.

  • akistotleakistotle Member 🍌🍌
    edited February 2018 9377 karma

    Hi @nc______ !

    I edited your title for formatting, but were you referring to PT1.S3.Q12, which starts with "Some of the most prosperous...," because (C) isn't the answer for the question. Can you double check? I'm so sorry if I mistakenly changed the number of the PT/Q! :cry:

    Thanks,

Sign In or Register to comment.